CSIR NET BIOLOGY (June - 2023 Sift 2)
Previous Year Question Paper with Solution.

21. In a cooperatively breeding species, under which condition is a helper more likely to exhibit philopatry?

(a) If adult survivorship of higher for group members than for solitary individuals

(b) When resources are abundant and widely distributed

(c) When the chance of acquiring territory is higher

(d) If the possibility of acquiring mates is higher outside the group

Ans. (a)

Sol. In cooperatively breeding species, helpers are individuals who do not breed but instead help to raise the young of their parents or other relatives. Helpers may do this for a variety of reasons, including increased survival rates, increased access to mates, and increased genetic fitness.

22. A tree species has leaves that contain an allelochemical compound that leaches into the soil and prevents the growth of its own seedlings. What kind of dispersion pattern is likely as a result of this process in the adult population of this species?

(a) Random

(b) Clumped

(c) Uniform

(d) Bimodal

Ans. (c)

Sol. Uniform dispersion is characterized often by organisms that are territorial in nature and wind up equally spaced apart due to the defense of their immediate surroundings. A uniform distribution tends to line up in even rows similar to trees planted In neat and even rows. An example of a plant that does this in nature is the sage plant {Salvia leucophylla). This sage plant releases a toxin that kills plants in the nearby vicinity. Since the area immediately surrounding the sage is made Inhabitable by the toxin, sage plants wind up aligning with a distinctly empty radius surrounding them.

23. Which one of the following statements is NOT true for a continuous culture-based fermentation?

(a) The exponential phase of growth is extended

(b) Nutrients are utilized efficiently and faster

(c) Risk of contamination is lower than batch fermentation

(d) A chemostat allows maintenance of growth rate during fermentation

Ans. (c)

Sol. In a continuous culture-based fermentation, the risk of contamination is generally higher compared to batch fermentation. This is because in continuous culture, the fermentation process is carried out for an extended period of time, often with a continuous influx of fresh nutrient medium and the continuous removal of waste products.

24. Which one of the following descriptions does NOT apply to circadian rhythmicity?

(a) A process that can be found in bacteria, plants, fungi and animals

(b) A process that is rhythmic only in the presence of 24 hour light and dark cycle

(c) A process that can be synchronized by environement cycles

(d) A process that can be disrupted by prolonged exposure to costant darkness

Ans. (b)

Sol. Circadian rhythmicity is a process that occurs in organisms on a roughly 24-hour cycle, even in the absence of environmental cues such as light and dark. This process is thought to be controlled by an internal clock.

It Is not limited to the presence of a 24-hour light and dark cycle. Circadian rhythms can be found in a wide range of organisms. Including bacteria, plants, fungi, and animals.

25. Which one of the following statements related to photosynthesis is NOT correct?

(a) Light reaction takes place in the thylakoid membranes

(b) ATP and NADPH are produced in thylakoid membranes

(c) Lumen is the enclosed interconnected region of the thylakoid membranes

(d) NADPH is produced during carbon reactions by the enzymes present in stroma

Ans. (d)

Sol. NADPH is actually produced during the light-dependent reactions, specifically in the thylakoid membranes of the chloroplasts.

26. Several proteins are modified by phosphorylation at specific amino acid residuces to alter their activities, Which one of the following amino acids is NOT typically a site of phosphorylation in proteins?

(a) Lysine

(b) Serine

(c) Threonine

(d) Tyrosine

Ans. (a)

Sol. Amino acids that can be phosphorylated are histidine, aspartate, serine, threonine, and tyrosine.

27. Which one of the following statements regarding ligand identity and effector dynamics is typically correct?

(a) Specificity of receptors do not determine effector dynamics

(b) Ligands and receptros do not cluster at the cell interface to select the effector

(c) Ligand concentration can be represented by the amplitude and duration of adaptive pulses of effector activity

(d) Effector dynamics does not depend on how many different types of ligands influence a single pathway.

Ans. (c)

Sol. In cellular signaling pathways, ligands bind to specific receptors, initiating a cascade of events that lead to effector activity. The concentration of ligands can modulate the activity of effectors, and this concentration is often represented by the amplitude (strength) and duration of adaptive pulses of effector activity.

28. Which one of the following does NOT characterize aging?

(a) An insulin/IGF-1 signalling system plays an important role in controlling lifespan.

(b) Lifespan increases due to resistance to oxidative stress

(c) Shortening of telomeres

(d) Female mice with a mutation in the IGF-1 and IGF-2 show reduced lifespan.

Ans. (d)

Sol. In animal models, down-regulation of the IGF-l/insulln system significantly prolongs the lifespan.

29. Select the correct former name for The International Code of Nomenclature (ICN) which was changed as part of the Melbourne code

(a) International Code of Zoological Nomenclature (ICZN)

(b) International Code of Nomenclature for Algae, Fungi, and Plants (ICNafp)

(c) International Code of Botanical Nomenclature (ICBN)

(d) International Code of Nomenclature for Cultivated Plants (INCP)

Ans. (c)

Sol. The International Code of Botanical Nomenclature (ICBN) was the former name for the code that governed the scientific naming of plants, algae, and fungi. However, it was replaced by the International Code of Nomenclature for algae, fungi, and plants (ICNafp) in the Melbourne Code, which was adopted in 2011.

30. Which one of the following synaptic vesicles (as observed under transmission electron microscope) contains catecholamines?

(a) Small, round shaped and clear

(b) Small, round shaped and dense core

(c) Large dense core

(d) Small, flattened and clear

Ans. (b)

Sol. Catecholamines are stored in small, round vesicles with a dense core. These vesicles are called synaptic vesicles. Synaptic vesicles are located in the presynaptic terminal of a neuron. When an action potential reaches the presynaptic terminal, It causes the release of catecholamines from the synaptic vesicles into the synaptic cleft.

31. Different experimental aproaches were used to quantify serum levels of IL-17 in human patient samples. Which one of the following approaches provides the most accurate quantification in a standard clinical setting?

(a) Sandwich ELISA, with monoclonal capture and detection antibodies against the same epitope of human IL-17

(b) Fractionation of the serum sample on SDS-PAGE followed by Western blotting with polyclonal anti-human IL-17 antibody

(c) Direct ELISA by coating the plate with patient serum and detection with polyclonal anti-human IL-17 antibody

(d) Sandwich ELISA with monoclonal capture and detection against different epitopes of human IL-17.

Ans. (d)

Sol. The sandwich ELISA technique is commonly used for quantifying proteins in biological samples. It involves the use of specific antibodies to capture and detect the target protein. In the case of IL-17 quantification, using monoclonal antibodies against different epitopes of human IL-17 in a sandwich ELISA provides higher specificity and sensitivity compared to other approaches.

32. Which one of the following leads to the induction of defensin PDF 1.2 in Arabidopsis?

(a) Wounding

(b) Salicylic Acid (SA)

(c) Dichyloroisonicotinic acid (INA)

(d) Ethylene

Ans. (d)

Sol. The induction of defensin PDF1.2 in Arabidopsis is indeed primarily regulated by ethylene. Ethylene is a gaseous plant hormone that plays a crucial role in various physiological processes, including defense responses to pathogens. When Arabidopsis plants are exposed to pathogenic microbes, ethylene production increases, leading to the induction of several defense-related genes, including defensin PDF1.2.

33. Which one of the following statements is correct?

(a) None of the virulence genes of Agrobacterium tumefaciens are expressed constitutively

(b) Integration of T-DNA with the nuclear genome of plant cells occurs only by homologous recombination

(c) Host plant genes do not play any role in Agrobacterium-mediated transfer of T-DNA into plant cells.

(d) Opines are a source of introgen for Agrobacterium cells

Ans. (d)

Sol. v/rA and v/rG genes are expressed constitutively.

In the plant cell nucleus, the single-stranded T-strand is converted Into double-stranded T-DNA and Integrated

Into the plant genome. Integration into the plant genome occurs at an apparently random position through nonhomologous recombination.

Opines are of different types: nopaline, octopine and agropine. These opines are condensation products of either an amino add and a keto add or an amino add and a sugar. The opines are used as a carbon and nitrogen source and are catabolized by Agrobacterium.

34. In the avian embryo, the bastocoel-like fluid-filled cavity is formed between

(a) epiblast and hypoblast

(b) hypoblast and yolk

(c) primary hypoblast and secondary hypoblast

(d) Koller's sickle and Posterior Marginal Zone

Ans. (a)

Sol. In avian embryos, the blastocoel-like fluid-filled cavity, known as the subgerminal cavity or sub-blastocoelic cavity, is formed between the epiblast and hypoblast layers. The epiblast is the upper layer of cells, while the hypoblast is the lower layer of cells in the early avian embryo. The formation of this fluid-filled cavity plays a crucial role in embryonic development and is involved in the establishment of the germ layers and subsequent tissue differentiation.

35. Which one of the following honeybee species is native to the Indian subcontinent?

(a) Apis melltfera

(b) Apis dorsata

(c) Apis koschevnikovi

(d) Apis nigrocincta

Ans. (b)

Sol. Apis dorsata, commonly known as the giant honeybee, is found in various regions of Asia, including the Indian subcontinent. They are known for their large size and distinctive behavior of building single comb nests, often attached to high tree branches or cliffs. Apis dorsata is an important species for honey production and pollination in the Indian subcontinent.

36. Which one of the following is NOT a characteristic feature of platelets present in human blood

(a) They are 2-4 µm in diameter

(b) They lack nuclei

(c) Their half-life is 20-24 days

(d) They are derived from bone marrow megakaryocytes

Ans. (c)

Sol. Platelet* (or thrombocyte*), are not whole cell* but. are cell fragment* derived from megakaryocyte*. A tingle mtgakaryocytt typically produce* about 1000 platelet* Megakaryocyte* are derived from the tame hemopoietic *tem cell* that give rite to the erythrocyte* and leukocyte* Platelet* are basically fragment* of megakaryocyte* in which small cytoplasm remain enclosed by a piece of the plasma membrane but they lack nuclei. The hormone thrombopoletln. produced by the liver, increases the number of megakaryocytes in the bone marrow and stimulate* megakaryocytes to produce more platelets Platelet* break off from th* megakaryocytes in rod bone marrow and then enter the blood circulation Between 150,000 and 400.000 platelet* are present in each microlitre of blood. Platelet* have a short life span, normally Just 5 to 9 day* Aged and dead platelets ar* removed by fl>*d macrophage* in the spleen and liver.

37. A plant beterozygous for a dominant trait was selfed. The progeny had 140 plants showing the dominant trait and 20 plants showing the recessive trait. A researcher hypothesised that there are two genes with identical functions that control the dominant trait. The researcher also proposed that the two genes are not linked. The researcher carried out a chi-square test to test the hypothesis. Which one of the following options is the correct chi-square value (rounded to second decimal) obtained by the researcher?

(a) 22.86

(b) 13.33

(c) 10.67

(d) 5.71

Ans. (c)

Sol. There are two genes with identical functions that control the dominant trait.

When a dlhybrid cross produces progeny in two phenotypic classes in a 15:1 ratio, this can be because the two loci's gene products have the same (redundant) functions within the same biological pathway.

Expected ratio in F2 generation : 15 :1

Dominant trait = 140

Recessive trait = 20

Total = 160

Expected : Dominant -15/16 × 160 = 150

Recessive - 1/16 × 160 « 10

Observed : Dominant – 140

Recessive – 20

2 = y (Observed – Expected)2 y(O – E)2

Observed (O) Expected (E) O-E (O-E)J/E

140 150 –10 100/ 150 = 0.67

20 10 10 100/ 10 = 10

X2 = 10+ 0.67 = 10.67

38. Which one of the following fossils is no longer considered to ta true vascular plant based on the structure of the secondary thickening of the conducting elements?

(a) Asteroxylon mackiei

(b) Lepidodendron licopodites

(c) Rhynia major

(d) Sphenophyllum plurifollatum

Ans. (c)

Sol. Rhynia major is a fossil plant that existed during the Early Devonian period. While it was initially considered to be a true vascular plant, recent studies have shown that the structure of its secondary thickening of the conducting elements is different from what is typically observed in true vascular plants. As a result, Rhynia major is no longer considered to be a true vascular plant.

39. Select the group of plants the are known to have an increase in the amount of vascular tissues by means of secondary growth from a vascular cambium

(a) gymnosperms only

(b) dicotyledons only

(c) dicotyledons and monocotyledons

(d) dicotyledons and gymnosperms

Ans. (d)

Sol. Dicotyledons are a group of flowering plants that typically have two cotyledons (seed leaves) and exhibit secondary growth, which leads to the formation of a vascular cambium. The vascular cambium is a meristematic tissue that produces secondary xylem (wood) towards the inside and secondary phloem towards the outside, thereby increasing the girth of the stem.

Gymnosperms, on the other hand, are a group of plants that includes conifers, cycads, and ginkgoes. They also exhibit secondary growth and possess a vascular cambium. The vascular cambium in gymnosperms produces secondary xylem and secondary phloem, allowing for an increase in the amount of vascular tissues.

40. Reduction in the frequency of heterozygous genotype with a concomitant increase in the freqency of homozygous genotype, in context of random mating is due to

(a) Genetic drift

(b) Intense inbreeding

(c) Revere mutation

(d) Founder effect

Ans. (b)

Sol. In most species, including all mammals, inbreeding is associated with a reduction in heterozygote frequencies (l.e., heterozygosity) and increase in homozygote frequencies (i.e., homozygosity).

41. To examine the in vivo co-localization pattern of two different proteins using fluorescently labeled antibodies, which one of the following combinations of fluorochromes will be appropriate?

(a) Alexa 488 and Cy5

(b) Alexa 488 and FITC

(c) Alexa 647 and Cy5

(d) Fluorecein and FITC

Ans. (a)

Sol. Alexa 488 and CyS are two different fluorochromes that emit light at different wavelengths. Alexa 488 emits light at a wavelength of 488 nm, while Cy5 emits light at a wavelength of 647 nm. This means that they can be used to label two different proteins and then distinguished from each other using a fluorescence microscope.

42. Which one of the following statements about phytochromone ABA is correct?

(a) High level of ABA predominantly promotes vivipary

(b) ABA--D-glucosyl ester in an active form of ABA

(c) Inactivation of ABA involves its oxidation to phaseic acid

(d) ABA biosynthesis occurs entirely in the plastids

Ans. (c)

Sol. Vivipary: ABA-deficient embryos) may exhibit precocious germination and vivipary. Vivipary is the germination of mature seed within the fruit on maternal (mother) plant prior to dispersal. It is rare in angiosperms and is largely restricted to mangroves where seeds germinate while attached to the mother plants and seedlings are shed, stick Into the mud below, and continue to grow. The phenomenon of seedling formation without completing normal embryonic development is called precocious germination. Inactivated ABA or low levels of ABA can lead to precocious germination and vivipary.

43. Introns in the eukaryotic genes are found in :

(a) rRNA and mRNA encoding genes but not in the tRNA encoding genes

(b) mRNA and tRNA encoding genes but not in the rRNA encoding genes

(c) mRNA encoding genes but not in the tRNA and rRNA encoding genes

(d) rRNA, tRNA and mRNA encoding genes

Ans. (d)

Sol. Intron type Where found

GU-AG Intron* Eukaryotic nuclear pre-mRNA

AU-AC intron* Eukaryotic nuclear pre-mRNA

Group 1 intron* Eukaryotic nuclear pre-rRNA, organelle RNA*. tome prokaryotic RNA*

Group II intron* Organelle RNAs, some prokaryotic RNAs

Pre-tRNA Intron* Eukaryotic nuclear pre-tRNA

Archaeal Intron* Various RNA*

44. A bacterial culture initiated from a single bacterial cell with a DNA repair-deficient system is ioculated ito several individual test tubes and allowed to grow in prallel. Wild type cells are also inoculated in a similar manner and grown simultaneously. After several generations, individual cultures are tested for resistance to antibiotics. Which one of the following statements describes the most likely outcome?

(a) More antibiotic resistant cell will emerge from the DNA repair-deficient cultures and all wild type cells will be sensitive

(b) Wild type cell will produce more antibiotic resistant populations than the DNA repair-deficient cells

(c) The DNA repair-deficient cells any produce more antibiotic resistant cells but wild type cells will alo produce some antibiotic resistant population

(d) The DNA rapair-deficient cells would be dead and therefore will not produce any resistant population of cells.

Ans. (c)

Sol. DNA repair-deficient cells are more likely to develop antibiotic resistance because they are more susceptible to DNA damage. When DNA is damaged, it can lead to mutations, which can then lead to antibiotic resistance. Wild type cells, on the other hand, have a more robust DNA repair system, which makes them less susceptible to DNA damage and therefore less likely to develop antibiotic resistance.

45. One gram of a polysaccharde composed of 1000 glucose units has the same effect on osmolarity as that of

(a) 1 mg glucose

(b) 100 mg glucose

(c) 500 mg glucose

(d) 1000 mg glucose

Ans. (a)

Sol. One gram of a polysaccharide comprised of 1,000 glucose units contains 1,000 glucose molecules. One milligram of glucose contains 1 glucose molecule. Therefore, both solutions have the same number of solute particles, and the same osmolarity.

46. In cell membranes, the lipid molecules are arranged as a continuous double layer, with an approximate thickness of

(a) 20 nm

(b) 50 nm

(c) 5nm

(d) 1nm

Ans. (c)

Sol. The lipid bilayer consists of two layers of lipid molecules with their hydrophilic (polar) heads facing outward and their hydrophobic (nonpolar) tails facing inward. This arrangement provides the structural integrity of the cell membrane and regulates the movement of molecules in and out of the cell. The thickness of the lipid bilayer is approximately 5 nanometers (nm).

47. Which one of the following methods is NOT useful for sampling pteridophytes to study their distribution patterns?

(a) Ad libitum sampling

(b) Quadrat sampling

(c) Belt transect sampling

(d) Random sampling

Ans. (a)

Sol. Ad libitum sampling is a type of sampling in which the researcher collects data on whatever organisms they happen to see. This type of sampling is not useful for studying the distribution patterns of pteridophytes because it is not a systematic approach. The researcher may not see all of the pteridophytes in an area, and they may not see them in proportion to their abundance.

48. Which one of the following curve correctly depicts the relationship of the NPP/GPP ratio with latitude?

(a) A

(b) B

(c) C

(d) D

Ans. (a)

Sol. The NPP to GPP ratio can vary with latitude due to differences in environmental factors, such as temperature, sunlight availability, and water availability. Generally, the NPP/GPP ratio decreases with increasing latitude.

49. Which one of the following statements regarding regeneration in Hydra is correct?

(a) It follows only stem cell-mediated regeneration

(b) It follows only stem cell-mediated regeneration and morphallaxis

(c) It follows stem cell-mediated regeneration, morphallaxis and epimorphnosis

(d) If follows only morphallaxis

Ans. (c)

Sol. Hydra's regeneration involves multiple mechanisms, including stem cell-mediated regeneration, morphallaxls, and epimorphosis. Morphallaxls is a type of regeneration hallmarked by the absence of cellular proliferation. The existing tissue is repatterned to replace the lost tissue. Epimorphosis is a type of regeneration involving active cell proliferation for the completion of regeneration.

50. In a lac operon, a nonsense mutation in the gene encoding beta-galactosidase was found to interfere with the expression of downstream permease and transacetylase genes. Which one of the following may explain this observation most appropriately?

(a) polar effect of the mutation

(b) trans-effect of the mutation

(c) Binding of the release to the nonsense codon prevents traslation of the downstream cistrons

(d) Formation of a stem-loop structure in the upstream cistron prevents translation of downstream cistrons

Ans. (a)

Sol. A mutation that affects the transcription or translation of part of the operon downstream of the mutant site is referred to as a polar mutation.

51. The flowering repressor gene that is responsible for the vermalization requirement in Arabidopsis is :

(a) Constans (CO)

(b) Flowering Locus D (FD)

(c) Flowering Locus T (FT)

(d) Flowering Locus C (FLC)

Ans. (d)

Sol. Mechanism of Moral induction in vernalized plant*: Genetic and physiological studies of the vernalization pathway In Arabidoptis have identified some of the genes that are involved in this process. Vernalization involves epigenetic changes in the expression of gene. FLC(flowering locus C). FLC encodes a MADS-box transcription factor that represses flowering.It is expressed predominantly in mitotically active regions. In Arabidopsis, FLC works bydirectly repressing the expression of FT gene in leaves and SOC1 and FD genes at the shoot apical meristem.

In response to vernalization, the amount of FLC mRNA and protein is reduced The reduction in FLC expression by vernalization involves chromatin remodeling of FLC that requires the V!N3 (vernalization Insensitive 3) protein. The products of two other genes. VRNl (VERNALIZATION!) and VRN2. are also responsible for repression of FLC gene but in a very different manner. Its products are needed for maintenance but not for initiation of FLC silencing. In this way vernalization promotes flowering by reducing FLC expression.

52. Which oe of the following options best represents the sequence of events leading to the phenomenon of introgression?

(a) only back crossing and hybridization

(b) hybridization, back crossing and stabilization

(c) stabilization, repeated hybridization

(d) hybridization, stabilization, back crossing, mutation

Ans. (b)

Sol. Introgression is the transfer of genetic material from one species or population into anotherthrough repeated backcrossing and hybridization.

53. Which one of the following options lists mechanism that drive ecological succession?

(a) Only facilitation and tolerance

(b) Disturbance and tolerance

(c) Only tolerance and inhibition

(d) Facilitation, tolerance and inhibition

Ans. (d)

Sol. J. H. Connell and R. O. Slayter (1977) outlined three different models to explain the ecological processes of community change during succession. These models are facilitation model, tolerance model and inhibition model.

54. Which one of the following statement is NOT a characteristic feature of aquaporins?

(a) They are integral membrane proteins in the major intrinsic protein (MIP) family

(b) They are absent in bacteria

(c) A highly conserved Asn-Pro-Ala (NPA) triad of residues is present in the N-terminal half of the protein

(d) A highly conserved Asn–Pro–Ala (NPA) triad of residues is present in the C-terminal half of the protein

Ans. (b)

Sol. Many animal and plant cells contain specialized water channel in their plasma membrane to faciltate the water flow called aquaporins. These are a family of transport proteins that allow water and a few other small uncharged molecules, such as glycerol, ammonia to cross membrane. Aquaporins (AQPs) assemble as homotertramers in which each monomer consists of six membrane-spanning α-helical segment with cytoplasmically oriented amino and carboxyl terminl. Two hydrophobic loops contain conserved asparagine-proline-alanine (NPA) motif for selectivity of the channel. Each monomer functions as an independent pore.

55. A colour blind father has a daughter who is also colour blind and has Turner's syndrome. The genotype of the daughter is due to

(a) Translocation event in the father

(b) Translocation event in the mother

(c) Non-disjunction event in the mother

(d) Non-disjunction event in the father

Ans. (c)

Sol. In this scenario, the father Is color blind, which suggests that he carries a mutation in the genes responsible for color vision. The daughter, being color blind like her father, would have inherited this mutation from him. The additional condition of Turner syndrome (45, X) In the daughter is not directly related to the father's color blindness. This condition occurs due to a non-disjunction event during meiosis in the mother, where the X chromosomes fail to separate properly.

56. Which one of the following statements about TATA Binding Protein (TBP) is NOT true?

(a) It is a component of transcription factor TFIID

(b) TBP recognize the TATA element by inserting one of its α-helices into the major groove of DNA

(c) The TBP-DNA interactions causes the DNA to bend

(d) The TBP-DNA interaction is governed in part by the intercalation of the side chains of phenylalanine residues of TBP between the base pairs at the two ends of the TATA element sequence

Ans. (b)

Sol. Genes transcribed by RNA polymerase 11, first bind with the general transcription factor, TFIID. It is involved In transcription from all polymerase II promoters and constitutes part of the basic transcription machinery. TFIID is a complex made up of the TATA-bmding protein (TBP) and at least 14 TBP-associated factors (TAFs). TBP is a sequence-specific protein that binds to DNA via its unusual TBP domain, which makes contact with the minor groove In the region of the TATA boxj. TBP uses an extensive region of p-sheet to recognize the minor groove of the TATA box. This is unusual. More typically, proteins recognize specific DNA sequences using a-helices inserted into the major groove of DNA. When TBP binds DNA, it causes the minor groove to be widened to an almost flat conformation; It also bends the DNA by an angle of about 80°. Many promoters do not contain a

57. Which one of the following parameters of a healthy leaf plays the major role in its reflectance in the near infrared region?

(a) Water content in the leaf

(b) Concentration of chlorophyll in the leaf

(c) Concentration of carotenes and xanthophylls in the leaf

(d) Arrangement of spongy and palisade mesophyll tissue of the leaf

Ans. (d)

Sol. The near-infrared region of the electromagnetic spectrum is strongly absorbed by water, so the reflectance of a leaf In this region Is Inversely proportional to its water content. This means that leaves with high water content will have low reflectance in the near-infrared region, while leaves with low water content will have high reflectance.

58. Based on the reported estimates of biodiversity in India, Select the correct option that represents the decreasing order of total number of species reported in these taxa.

(a) Angiosperms > Insects > Algae > Birds > Fishes > Mammals

(b) Insect > Angiosperms > Algae > Fishes > Birds > Mammals

(c) Algae > Insects > Angiosperms > Birds > Fishes > Mammals

(d) Insects > Algae > Angioperms > Birds > Mammals > Fishes

Ans. (b)

Sol. Insects > Angiosperms > Algae > Fishes > Birds > Mammals. According to available estimates, insects have the highest number of reported species in India, followed by angiosperms (flowering plants), algae, fishes, birds, and mammals.

59. Which one of the following class of plant secondary metabolites in present specifically in the order Brassicales?

(a) Glucosinolates

(b) Alkaloids

(c) Phenolics

(d) Terpenoids

Ans. (a)

Sol. Glucosinolates are a group of secondary metabolites found in plants, particularly in the 8 rassicaceae family. These compounds are known for their pungent and bitter taste. Glucosinolates play an important role in plant defense against herbivores, pathogens, and Pests.

60. Which one of the following statements is TRUE?

(a) A, B and Z DNA helices are left-handed

(b) A, and B DNA helices are right-handed, Z DNA helix is left-handed

(c) A, and Z DNA helices are left-handed, B DNA helix is right-handed

(d) A and B DNA helices are left-handed, Z DNA helix is right-handed

Ans. (b)

Sol. DNA can exist in different forms, and the two most common forms are the B-form and the Z-form. The B-form DNA helix, which is the most common and biologically relevant form, is right-handed. Similarly, the A-form DNA helix is also right-handed. Both the A-form and B-form DNA helices are commonly found in nature.

61. In response to a Wnt singal , -catenin enters the nucleus and binds to the LEF1/TCF proteins by displacing which one of the following proteins?

(a) CBP

(b) GSK3

(c) Groucho

(d) NEMO

Ans. (c)

Sol.

62. Which one of the options given below is NOT desirable when setting up nature reserves in the tropics?

(a) Reserves that are linked to each other by corridor

(b) Reserves that are surrounded by a buffer zone of same ecosystem

(c) High edge-to-area ratio of the reserve

(d) Circular shaped reserve

Ans. (c)

Sol. An edge-to-area ratio refers to the proportion of the boundary between the reserve and surrounding areas to the total area of the reserve. A high edge-to-area ratio indicates a reserve with a large perimeter relative to its core area.

63. Which one of the following is NOT typically a product of fermentation?

(a) Cheese

(b) Black tea

(c) Kombucha

(d) Green tea

Ans. (d)

Sol. Green tea is not typically a product of fermentation. It is produced by harvesting and drying the leaves of the Camellia sinensis plant. The leaves undergo minimal oxidation during processing, which helps to retain the natural green color and flavors of the tea. This minimal oxidation process differentiates green tea from other types of tea, such as black tea, which undergoes full oxidation.

64. A cruciform structure of chromosomes during meiosis is a characteristic feature of :

(a) Translocation

(b) Inversion

(c) Deletion

(d) Duplication

Ans. (a)

Sol.

65. In eukaryotic genes DNA sequences that define gene promotes occur:

(a) only in the regions upstream of the transcription start sites

(b) only in the regions that represent the transcribed parts of the genes

(c) only in the regions downstream of the transcription termination sites

(d) either in the regions upstream of the transcription start site or within the transcribed regions of the gene

Ans. (d)

Sol. In eukaryotes, the term promoter is used to describe all the sequences that are important in the initiation of transcription of a transcription unit. For some transcription units, these sequences not only include the core promoter (sometimes also called the basal promoter), which is the site at which the initiation complex is assembled, but also one or more upstream regulatory promoter elements which, as their name implies, lie upstream of the core promoter and regulates transcription. A core promoter is typically 40-60 base pairs long, extending either upstream or downstream from the transcription start site. Initiation of transcription in eukaryotes requires the enzyme RNA polymerase and transcription factors. The transcription factors, rather than the RNA polymerase, are principally responsible for recognizing the promoter. This is different from the bacterial RNA polymerase, where it is the RNA polymerase that recognizes the promoter sequences. The transcription factors create a structure at the promoter to provide the target that is recognized by the RNA polymerase.

66. In a hydrogen bond of the type D-H...A, where D-H is a weakly acidic donor group and A is a lone-pair-bearing acceptor atom, the D...A distance is

(a) one-and-a-half times the sum of the van Der Waals radii

(b) equal to the sum of the van Der Waals radii

(c) less than te sum of the van Der waals radii

(d) twice the sum of the van Der Waals radii

Ans. (c)

Sol. The D...A distance in a hydrogen bond is typically less than the sum of the van der Waals radii of the D-H and A atoms. This Is because the hydrogen bond creates a partial negative charge on the acceptor atom and a partial positive charge on the hydrogen atom. The partial charges on the atoms repel each other, which increases the distance between the atoms.

67. The ColEI plasmid has a low to medium copy number. However, pUC18, which is also a ColE1-based plasmid, has a high copy number because :

(a) It has a mutation in RNAI (antisense RNA) and does not carry the rop gene

(b) Ith has a mutation in RNAII (primer for replication initiation) and does not carry the rop gene

(c) It has a mutation in RNAI and the rop gene is overexpressed

(d) It has a mutation in RNAII and the rop gene is overexpressed

Ans. (b)

Sol. The pUC18 plasmid, which is a ColE1-based plasmid, has a high copy number compared to the ColE1 plasmid. This is due to specific mutations and gene expression characteristics. The mutation in RNAJI, which serves as the primer for replication initiation, allows for more efficient initiation of plasmid replication. This leads to an increased number of plasmid copies within the bacterial cell.

68. Which one of the following is NOT a vasoconstrictor?

(a) Prostacyclin

(b) Thrombhoxane A2

(c) Angiostensin-II

(d) Endothelin-I

Ans. (a)

Sol. Prostacyclins (PGI2) is generated from PGH2 by a enzyme prostacyclin synthase that is particularly abundant in endothelial cells. Prostacyclin serves as a potent a potent vasodilator and inhibitor of platelet aggregation. It also promotes differentiation and inhibits proliferation in vascular smooth muscle cells.

69. Which one of the following mechanisms permits immuoglobulin to be synthesized in either a membrane-bound or secreted form?

(a) Allelic exclusion

(b) Class switch recombination

(c) Differential RNA processing

(d) Codominant expression

Ans. (c)

Sol. Cells switch from the synthesis of the membranebound to secreted forms of igM and igD by alternative RNA splicing. Whether secreted or membrane-bound antibody Is made depends on which poly(A) site is selected during processing of the primary transcript. The ability to switch between the membrane-bound and the secreted form of Immunoglobulin heavy chains is controlled by alternative use of polyadenylation sites (not by alternative splicing).

70. Which one of the following conditions associated with chromosome 15 may cause Prader-Willi syndrome?

(a) Paternal uniparental disomy

(b) Maternal uniparental disomy

(c) Imprinting of 15q11-q13 locus in maternal copy

(d) Imprinting of 15q23-q25 locus in paternal copy

Ans. (b)

Sol. Prader-Willi syndrome (PWS) is a genetic disorder associated with chromosome 15. It occurs when there is a lack of expression of specific genes on the paternal copy of chromosome 15. The most common genetic mechanisms leading to Prader-Willi syndrome are deletions on the paternal chromosome 15 or maternal uniparental disomy (UPD).

71. An investigator identified a nuclear localizatio signal (NLS; Pro-Lys-Lys-Lys-Arg-Lys) at theC=terminus of the protein X (50 kDa). To analyse the localization of protein X, the investigator fued protein X with GFP at the C-terminus. The fusion protein was detected in the cytosol. When the nuclear localization signal was fused with GFP at the N-terminus, the NLS-tagged GFP extensively localized in the nucleus. Based on this observation, the invetigator made a few hypotheses:

A. The basic amino-acid stretch in the protein X-GFP chimeric construct is masked by the GFP sequence and thus not capable of directing entry of protein in G-GFP into the nucleus

B. The X-protein in the full-length X-GFP chimeric protein is post-translationally modified that impacts its import into the nucleus

C. Fusion with GFP makes the protein X too bulky to enter the nucelus through the nuclear pore complex

D. The GFP is post-translationally modified that impacts the import of protein X-GFP into the nucleus

Which one of the following options gives the combination of all possible hypotheses that best explain Protein-X's trafficking mechanisms?

(a) A and D

(b) B only

(c) A and B

(d) C and D

Ans. (c)

Sol. A. The basic amino acid stretch in the protein X-GFP chimeric construct is masked by the GFP sequence and thus not capable of directing entry of protein X-GFP into the nucleus. This suggests that the fusion of GFP at the C-terminus may be interfering with the nuclear localization signal (NLS) of protein X, preventing it from directing the protein to the nucleus. B. The X-protein in the full-length X-GFP chimeric protein is post-translationally modified, which impacts its import into the nucleus. This hypothesis suggests that some post-translational modifications occurring in the full-length X-GFP chimeric protein may be affecting its ability to enter the nucleus.

72. Follwoing statements are made about the chemical properties and distributions of the respiratory pigments found in animals :

A. Hemoglobins are the most common and widespred respiratory pigments in vertebrates and invertebrates and are always present in blood cells.

B. The heme structure in hemoglobins is an iron (ferrous) porphyrin which varies widely among species,. and also varies among the different molecular forms of hemoglobin within any single species. The globin, however is exactly identical.

C. Hemocyanin contains are similar to hemocyanin, but have a lower affinity for oxygen binding than hemocyain present in blood cells of some marine annelid worms.

D. Chlororuorins are similar to hemocyanin, but have a lower affinity for oxygen binding than hemocyanin present in blood cells of some marine annelid worms

E. Hemerythrins are non-heme, iron-containing repiratory pigment that have a limited and scattered distribution.

Which one of the following options represents the combination of all Incorrect statements?

(a) A, B and C

(b) A, B and D

(c) B, D and E

(d) C, D and E

Ans. (b)

Sol.

73. The figure below represent the fundamental and realised niche of two species.

Which one of the following options correctly identifies the fundamental niche and realised niche of any one of the species?

(a) Fundamental niche– P; Realised niche – Q

(b) Fundamental niche– Q; Realised niche – P

(c) Fundamental niche– P; Realised niche – R

(d) Fundamental niche– R; Realised niche – P

Ans. (b)

Sol.

74. Blood hemostasis is the interplay of several intrinsic and extrinsic factors. Deficiency of some of the blood clotting factors and their clinical manifetations are listed below :

Which one of the following options represents all correct matches?

(a) A-(i), B-(ii), C-(iii), D-(iv)

(b) A-(iv), B-(ii), C-(iii), D-(i)

(c) A-(ii), B-(iii), C-(iv), D-(i)

(d) A-(iii), B-(i), C-(i), D-(iv)

Ans. (b)

Sol.

75. You are sampling birds in a forest community to determine species diversity of birds in this region. How would you assess the sampling effort to ensure that you have obtained a reasonable estimate of the diversity in the region?

(a) Based on the species accumulation curve

(b) You cannot determine this, as sampling effort and species richness are independent of one another

(c) Based on the calculation of Morisita-Horn similarity index

(d) Based on the calculation of Simpon's diversity index

Ans. (a)

Sol. A specie* accumulation curve (or collector's curve) is used to estimate the number of different species (l.e. species richness) In a particular area. It is a graph which records the cumulative number of species in a particular area observed or collected during the survey.

During the process of data collection, additional species are added to the pool of all previously observed or collected species. Lot us take example to understand this. Suppose, we have to measure the total number of different tree species in an area. For this, we first delineate the area where the estimate of species richness is to be made. After that, a number of smaller sample areas (plots) within the large delineated area are to be selected, and a survey is conducted within the sample areas (plots).

76. The following statements are made about post-transcriptional processing

A. RNA editing can occur via the deamination of cytosine residues, leading to formation of uracil and thus a change in coding sequence.

B. The major spliceosomal complex mediates than removal of Group II introns

C. Trans-splicing events seen in trypanosomes allow the formation of multiple gene products by bringing together different combinations of exons of three or more genes

D. Capping of eukaryotic mRNAs occurs exclusively in the nucleus of the cell.

Which one of the following options represents the combination of all correct statements?

(a) A and D

(b) B and D

(c) B and C

(d) A only

Ans. (d)

Sol. Functions of S'-cap: The 5'-cap Is an evolutionary conserved modification of eukaryotic mRNA. The 5 -cap serves as a unique molecular module that recruits cellular proteins and mediates cap-related biological functions such as pre-mRNA processing, protecting mRNA from degradation, nuclear export and cap-dependent protein synthesis. It was previously thought that 5 -capplng occurs exclusively In the nucleus, but RNA capping has been reported in the cytoplasm of mammalian cells and Trypanosomes also.

77. Pyruvate generated by glycolysis, is converted to acetyl-coenzyme A, which is metabolized by the citric acid cycle generating energy-rich molecules. From the choice given below, select the right combination of these molecules produced form one molecule of acetyl-CoA.

(a) 2 NADH + 2 FADH2 + 1 GTP

(b) 3 NADH + 1 FADH2 + 1 GTP

(c) 3 NADH + 1 GTP

(d) 4 NADH + 1 FADH2 + 1 GTP

Ans. (b)

Sol. Acetyl-CoA 3 × NAD* + FAD + NTP + 3 × H2O – a 2 × CO2 + 3 × NADH + FADH2 + NTP +

78. Below are some of the proposed roles of reactive oxygen species (ROS) in plant defense.

A. H2O2 may be directly toxic to pathogens

B. In presence of iron H2O2 gives rise to an extremely reactive hydorxyl radical

C. H2O2 leads to induced biosynthesis of salicylic acid (SA)

D. H2O2 production is always delayed during incompatible interactions.

Which one of the following options represents the combination of all correct statements?

(a) A, B and C

(b) B, C and D

(c) A, C and D

(d) A, B and D

Ans. (a)

Sol. Statement A is correct as hydrogen peroxide (H2O2) can exhibit direct toxic effects on pathogens.

Statement B is correct as in the presence of iron, H2O2 can generate hydroxyl radicals, which are highly reactive and can cause damage to pathogens.

Statement C is correct as H2O2 can induce the biosynthesis of salicylic acid (SA), which is involved in plant defense responses.

Statement D is not included in the correct combination as it states that H2O2 production is always delayed during incompatible interactions, which is not necessarily true.

79. The following are some statements made regarding mutations:

A. Change of DNA sequence from AGC to ATC in non-coding strand can have a major impact on the protein production

B. Suppressor mutation restores the original phenotype, only when a second mutation occurs at the original site of the mutation

C. Mutation rates remain the same in the all organisms

D. Strand slippage during replication is a consequence of loop formation in one strand of DNA

E. Hydroxylamine adds a hydroxyl group only on cytoine

Which one of the following options represents the combination of all incorrect statements?

(a) A and B

(b) B and C

(c) C and D

(d) D and E

Ans. (b)

Sol. Mutation rates can vary significantly among different organisms, as well as within different regions of the genome within a single organism. Several factors Influence mutation rates, including the type of organism, the specific DNA sequence or genomic region being considered, environmental factors, and evolutionary pressures.

A suppressor mutation is a second mutation that restores a function lost by the first mutation. Mutations of this kind are called suppressor mutations because they suppress the effects of the first mutations. True back mutation restores the original wild-type nucleotide sequence of the gene, whereas a suppressor mutation does not. Suppressor mutations may occur at distinct sites In the same gene as the first mutation or in different genes, even on different chromosomes.

80. In a tissue, cells are bund together by physical attachment between cell to cell or between cell to the exyracellualar matrix. The following are some of the characteristics of cell junctions :

A. Tight junctions are cell-cell junctions connecting the intermediate filament in one cell with that in the adjacent cell

B. Desmosmes are cell-matrix anchoring junctions connecting actin filament in one cell to the extracellular matrix

C. Gap junctions are channel-forming junctions allowing the passage of small water-soluble molecules from cell to cell.

D. Tight junctions are occulding junctions that seal the gap between two cells

E. Hemidesmosomes are cell-matrix anchoring junctions connecting intermediate filaments in one cell to the extracellular matrix

Which one of the following options represents the combination of All incorrect statements?

(a) A and B

(b) B and D

(c) C and E

(d) D and E

Ans. (a)

Sol.

81. The following experiment was designed to establish the synergy of Bcl2 with genes like Myc in leading to B-cell lymphormas

Identify the figure that correctly represents conditions under which mice succumbed to lymphomas.

(a)

(b)

(c)

(d)

Ans. (c)

Sol. When mice succumb to lymphomas, it means that they have developed and ultimately died from lymphatic cancer. Lymphomas are a type of cancer that affects the lymphatic system, which is an important part of the body's immune system. Lymphomas occur when there is an uncontrolled growth of lymphocytes, a type of white blood cell, leading to the formation of tumors in the lymph nodes or other lymphoid tissues.

82. The aminoacyl-tRNA synthesis (AARSs) in an organism have evolved to catalyze aminoacylation of their cognate tRNAs

(a) either at the 3'-OH or 2'-OH positions of the adenosine at the CCA end

(b) only at the 3' -OH position of the adenosine at the CCA end

(c) only at the 2'-OH position of the adenosine at the CCA end

(d) only at the Cl' position of the adenosine at the CCA end

Ans. (a)

Sol. Attachment of an amino acid to tRNA involves a covalent linkage between the carboxyl group of the amino acid and the 2' or 3'-hydroxyl group of the adenine containing nucleotide at the 3' end of the tRNA. This process (called amlnoacylatlon) is catalyzed by an enzyme called amlnoacyl-tRNA synthetase (also known as aminoacyl-tRNA ligase). Amlnoacyl-tRNA synthetases activate the ammo acids by covalently linking it to tRNA. When a tRNA is charged with the ammo add corresponding to its anticodon, it is called aminoacyl-tRNA. With a few exceptions, organisms have 20 aminoacyl-tRNA synthetases, one for each amino acid. Each of the 20 different synthetases recognizes one amino acid and all its compatible (or cognate) tRNAs. The 20 aminoacyl-tRNA synthetases fall into two distinct groups - Gass I and Gass U. Class 1 enzymes attach the amino acid to the 2'-OH group of the terminal nucleotide of the tRNA, whereas class II enzymes attach the ammo acid to the 3 -OH group. Gass 1 aminoacyl-tRNA synthetases are mostly monomeric, whereas class II aminoacyl-tRNA synthetases are mostly dimeric or multimeric.

83. When budding yeast (a facultative anaerobe) is grown for a few days in medium containing high glucose it show a growth pattern with two lag phases (see figure below)

Which one of the following statements best explains this growth pattern?

(a) In first lag phase, cells become acclimatized to the new glucose environment, in the second lag phase they undergo selective cell death and robust cells start dividing again

(b) In the second lag phase yeast cells switch from fermentation to utilizaing non-fermentatble carbon sources and the lag is to acclimatize to this souce of energy

(c) Yeast cells use glucose in the first exponential phase and use scrose in the second phase

(d) Yeast cells switch from mitotic to meiotic division in low glucose and hence require the lag phase to prepare for meiosis.

Ans. (b)

Sol. In the second lag phase, the yeast cells switch from fermentation to oxidative metabolism, where they start utilizing non-fermentable carbon sources such as ethanol or other metabolites. This transition requires the cells to acclimatize to the new energy source and activate the necessary metabolic pathways for oxidative metabolism. The lag phase allows the cells to adjust their metabolic machinery, enzyme expression, and energy production to efficiently utilize the non-fermentable carbon sources, eventually leading to renewed growth.

84. The following statements are made regarding cytokinin (CK) biosynthesis in plants "

A. Trans-zeatin (tZ) and iso-peptenyladenile (iP) are common active form of ioprenoid CKs.

B. CKs are present as nucleoside and glycosidic conjugates but not a nucleotide conjugates

C. Dephosphorylation and deribosylation steps are involved in two-step pathway for active CK formation

D. Lonely Guy (LOG) enzyme is involved in CK metabolism

Which one of the following options represents the combination of all correct statements?

(a) A, B and C

(b) A, B and D

(c) A, C and D

(d) B, C and D

Ans. (c)

Sol. Statement A is correct. Trans-zeatin (tZ) and isopentenyladenine (iP) are indeed common active forms of isoprenoid cytokinins (CKs) in plants. Statement B is incorrect. CKs can be present as nucleoside, glycosidic, and nucleotide conjugates. They can undergo different modifications and conjugation reactions in plants. Statement C is correct. The biosynthesis of active CKs involves a two-step pathway that includes dephosphorylation and deribosylation steps. Statement D is correct. The Lonely Guy (LOG) enzyme is indeed involved in cytokinin metabolism. It plays a role in the activation and degradation of CKs.

85. Following figure shows the early interactions between the Apical Ectopdermal Ride (AER) and the limb bud mesenchyme.

The red lines with block head indicate repression while th black lines indicate activation

The following statements were made regarding the development of a tetrapod limb :

A. When the limb bud grows Shh creates a new signalling centre that induces the posterior anterior polarity

B. When the concentration FGFs rises, it can inhibit Gremlin thus allowing MBPs to begin repressing the AER-FGFs.

C. FGFs 4, 9 and 17 from the AER inhibit Shh to stabilize the ZPA

D. Repression of Gremlin synthesis helps maintain the AFR.

Which one of the following options represents the combination of all correct statements?

(a) A and B

(b) A and C

(c) B and D

(d) C and D

Ans. (a)

Sol. Shh is a key signaling molecule involved in limb development. It is secreted by the zone of polarizing activity (ZPA) located at the posterior part of the limb bud. FGFs (Fibroblast Growth Factors) and BMPs are both involved in the development and maintenance of the apical ectodermal ridge (AER), which is a critical structure in limb development.

86. How long should it take the polypeptide backbone of a 6-residue, 10-residue, 15-residue and 20-residue folding nucleus to explore all its possible conformations? Assume that the polypeptide backbone randomly reorients every 10–13 seconds (s).

(a) 10–7s, 10–3s, 102s, 107s, respectively

(b) 10–10s, 10–6s, 103s, 1010s, respectively

(c) 10–5s, 10–2s, 10s, 103s, respectively

(d) 1s, 10s, 100s, 107s, respectively

Ans. (a)

Sol. Time t, in seconds, required for the protein to explore all the conformations t =

Assume that the polypeptide backbone randomly reorients every 10–13 seconds (s).

87. Given below are the list of plant derived alkaloids and their uses in modern medicine

Which one of the following options represents all correct matches?

(a) A-(iii), B-(iv), C-(ii), D-(i)

(b) A-(iii), B-(iv), C-(i), D-(ii)

(c) A-(iv), B-(i), C-(iii), D-(ii)

(d) A-(iv), B-(iii), C-(ii), D-(i)

Ans. (d)

Sol.

88. The effects of stimulation of cholineregic vagal fibers on the pacemaker potential of the cells of sinoatrial (SA) node of heart and on the nodal impulse generation are suggested below :

A. The nodal cell membrane becomes depolarized

B. The slope of the pacemaker potential is increased

C. The K+ condujctance of nodal cell membrane is decreased

D. The depeolarizing effect of 'h' current (Ih) on the membrane potential is slowed down due to the opening of G protein gated K+ channels

E. The opening of Ca++ channels are slowed down due to the decreased cAMP level in the nodal cells.

Which one of the following options represents the combination of all correct statements?

(a) A and B

(b) B and C

(c) C and D

(d) D and E

Ans. (d)

Sol. The depolarizing effect of the Ih current on the membrane potential is slowed down due to the opening of G protein-gated K+ channels. E. The opening of Ca2+ channels is slowed down due to the decreased cAMP level in the nodal cells.

89. The enzyme-catalysed reaction shown below follows Michaelis-Menten kinetics.

k1 = 1 × 108 M–1 s–1, k–1 = 4 × 104 s–1, k2 = 8 × 102 s–1

From the information given above, calcualte Km and Ks.

(a) Ks : 400 M–1 s–1, Km : 408 M

(b) Ks : 400 µM, Km : µM

(c) Ks : 400 µM s–1, Km : 408 µM

(d) Ks : 400 µM, Km : 408 µM

Ans. (d)

Sol.

If k2 is much smaller than k–1, the Michaelis constant can be expressed as

90. Given below are the list of the some of the most rare species on our planet (Column X) and the regions of the world where they occur (Column Y).

Which one of the following options represents all correct matches between Column X and Column Y?

(a) A-ii, B-i, C-iv, D-iii

(b) A-iii, B-iv, C-i, D-ii

(c) A-iv, B-iii, C-ii, D-i

(d) A-ii, B-iv, C-i, D-iii

Ans. (a)

Sol. Saola (Pseudoryx nghetinhensis): The saola is a critically endangered ungulate species found in the Annamite Range of Vietnam and Laos.

The Ili Pika is a small mammal species belonging to the pika family and is native to the Tianshan Mountains of Northwestern China.

The Greater Bamboo Lemur is an endangered primate species endemic to the eastern rainforests of Madagascar.

He Addax is a critically endangered antelope species found in the Sahara Desert of North Africa, specifically in countries like Niger, Chad, and Mauritania.

91. Shown below is the proton coupled carbon-13 NMR spectrum of sodim trimethylsilylpropanesulfonate (DSS), a common internal chemical shift standard spectroscopic studies of proteins and peptides. Also shown on the spectrum is the structure of DSS in which the different carbon atoms have been labeled a-f. The peaks in the NMR have been labeled 1-4.

Which of the following represents the correct assignments for the carbons in DSS?

(Hint – The nuclear spin quantum numbers of 1H and 13C are I = 1/2)

(a) peak 1–carbon a, peak 2-carbon b, peak 3–carbon c, peak 4–carbons d, e and f

(b) peak 1–carbon f, peak 2-carbon e, peak 3–carbon d, peak 4–carbons a, b and c

(c) peak 1–carbon d, peak 2-carbon b, peak 3–carbon f, peak 4–carbons a, b and c

(d) peak 1–carbon e, peak 2-carbon f, peak 3–carbon d, peak 4–carbons a, b and c

Ans. (b)

Sol. Peak 1 - carbon f b. Peak 2 - carbon e b. Peak 3 - carbon d b. Peak 4 - carbons a, b, and c

92. The figure below depicts the evolutionary tree of organisms based on characteristics that are depicted as numbers (i-iv).

Choose the option that correctly matches the characteristics to the numbers :

(a) (i)-Amniotic egg (ii)-Oviparous (iii)-Fur present (iv)-Tetrapod

(b) (i)-Oviparous (ii)-Amniotic egg (iii)-Fur present (iv)-Tetrapod

(c) (i)-Fur present (ii)-Oviparous (iii)-Tetrapod (iv)-Amniotic egg

(d) (i)-Tetrapod (ii)-Amniotic egg (iii)-Oviparous (iv)-Fur present

Ans. (d)

Sol. Number IV corresponds to Fur present, indicating the presence of fur or hair in the organism.

93. The two columns given below lists various organisms and their dispersal and distribution status in India.

Which one of the following options represents all correct matche between the above two columns?

(a) A-(i), B-(iv), C-(ii), D-(iii)

(b) A-(iii), B-(ii), C-(iv), D-(i)

(c) A-(iv), B-(i), C-(iii), D-(ii)

(d) A-(ii), B-(iii), C-(i), D-(iv)

Ans. (c)

Sol. Tabebuia rosea, commonly known as the pink trumpet tree or rosy trumpet tree, is a species of flowering tree native to Central and South America. Achatina fulica, also known as the giant African land snail, is a species of large snail native to East Africa.

Datura innoxia, commonly known as angel's trumpet or moonflower, is a species of flowering plant in the nightshade family.

Merops viridis, also known as the blue-throated bee-eater, is a species of bee-eater bird found in parts of Asia, including the Indian subcontinent and Southeast Asia.

94. The number of individuals of different specie in two communities P and Q is given below:

Based on the given data, select the correct statement?

(a) Community P has higher species diversity than Q

(b) Community Q has higher species diversity than P

(c) Both communities P and Q are equally diverse

(d) Data is not sufficient to compute species diversity

Ans. (b)

Sol.

95. Males of a species of grasshopper produce loud cell to attract femles. Mot energy of these cells lie in the species freqfiuency, while other frequencies have much less energy. This is depicted in a power spectrum (plots with solid line in the figures below). Females find males by listening to and recognizing the species-specific cell, and they are most sensitive to the species specific frequency. This is depicted using hearing threshold curves (plots in dashed lines in the figures below) This allows females to find even the softest males of their own species and ignore even the loud callers of other species, resulting, in reproductive isolation.

Which one of the following figures represents the correct option for the hearing threshold (dashed lines) of female, given the power spectrum (solid lines) of male calls of this grasshopper species?

(a)

(b)

(c)

(d)

Ans. (a)

Sol. The combination of the species-specific call produced by males and the high sensitivity of females to the corresponding frequency ensures reproductive isolation. Females can effectively locate and respond to even the softest calls produced by males of their own species. At the same time, they are less responsive to the calls of other species, even if those calls are louder or more prominent in other frequency ranges

96. Following statements are made regareding amphibian development

A. Fibronectin plays in important role in enabling the mesodermal cells to migrate into the embryo

B. Organizer secrete proteins that block the BMP signal, which allows the ectodermal cells to become epidermis

C. Wnt signalling causes a gradient of β-catenin along te anterior posterior axis of the neural plate, which appears to specify the regionalization of the neural tube

D. The more ventral blastomeres in the endoderm have high expression of nodal-related proteins

Which one of the following options represents the combination of all correct statements?

(a) A and B

(b) A and C

(c) B and C

(d) C and D

Ans. (b)

Sol. Fibronectin is an extracellular matrix protein that plays a crucial role in cell adhesion and migration. During amphibian development, mesodermal cells need to migrate and properly position themselves within the developing embryo.

Wnt signaling causes a gradient of â-catenin along the anterior-posterior axis of the neural plate, which appears to specify the regionalization of the neural tube. Wnt signaling is a key signaling pathway involved in many aspects of embryonic.

97. The following statements were made about X-chromosome inactivation in humans :

A. Meternally-derived X-chromosome has a greater chance of becoming inactivated in any given cell

B. Both X-chromosomes are activated during the process of oogenesis

C. The XIST gene encodes for a single, long non-coding transcript, which binds with the X chromosome and helps in its inactivation

D. The XIST gene expression is required to initiate inactivation of X-chromosome, and also to maintain inactivation form one cell generation to the next.

E. Tsix transcription affects the abundance of Xist RNA in cis.

Which one of the following options represents the combination of all correct statements?

(a) A, C and D

(b) B, C and E

(c) A and C only

(d) B and E only

Ans. (b)

Sol. The term 'antisense' refers to the fact that TSIX is complemetary in sequence to XIST. TSIX is also a long non-coding RNA (40 kb). Like XIST, TISX only acts on the chromosome that produces it i.e., cis-acting. Moreover, there is a inverse relationship between TSIX ad XIST expression. When TSIX transcription is reduced on one x, XIST expression increases and leads to inactivation of that same X-chromosome. In contrast, overexpression of TSIX prevents an increase in XIST expression on the same X-chromosome. The mechanism for triggering expression of the XIST gene from only one chromosome is incompletely understood. It appears that XICs on the two X-chromosomes sense each other and pair. The transient pairing results in transcription of XIST in one chromosome and TSIX on the other.

X-chromosome inactivation

There are two type of X-chromosomes inactivation–random X-inactivation and imprinted X-inactivation. In random X-inactivation, inactivation of either the materal or the paternal X-chromosome take place, Out of two X-chromosomes in the female cell, one comes from the male gamete (paternal XP) and the other from the female gamete (maternal, Xm). During random inactivation of X-chromosomes each cell first counts its number of X-chromosomes, then randomly chooses one X-chromosomes to remain active and silences the second X-chromosome. The random inactivation of Xn and Xn in somatic cells during early embryonic development is known as Lyon hypothesis.

98. The following statements refer to the observations made by a student, upon using 2, 6-dichloroisonicotinic acid (INA) to induce systemic acquired resistance (SAR) in tobacco. INR treatment.

A. enhances salicylic acid concentration in plants

B. Does not enhance salicylic acid concentration in plants

C. fails to activate SAR in nahG-expressing plants

D. activates SAR in nahG-expressing plants

Which one of the following options represents the combination of all correct statements?

(a) A and C

(b) A and D

(c) B and C

(d) B and D

Ans. (d)

Sol. Systemic acquired resistance (SAR) Is a defense mechanism observed in plants that enables them to enhance their resistance against a broad spectrum of pathogens, such as viruses, bacteria, and fungi. SAR is a form of Induced plant Immunity that occurs following an

Initial exposure to a pathogen or a signaling molecule released by the pathogen. 2,6-dichloroisonicotlnlc acid (INA) Is a synthetic compound that is known for its ability to Induce systemic acquired resistance In plants. It Is a chemical analog of the natural plant hormone called salicylic add (SA), which plays a crucial role In plant defense responses.

99. Given below is the list of viruses in Column X and their receptors in human host cells in Column Y.

Which one of the following options gives all correct matches between Column X and Column Y?

(a) A-i, B-ii, C-iii, D-iv, E-v

(b) A-v, B-iii, C-i, D-iv, E-ii

(c) A-ii, B-iii, C-v, D-iv, E-i

(d) A-v, B-iv, C-iii, D-ii, E-i

Ans. (c)

Sol.

100. A founder population has an An heterozygous genotype with a frequency of 1 and no individual with either AA or aa genotypes. With repeated self-fertilization, the frequency of AA, Aa and aa after tree generations will be :

(a)

(b)

(c)

(d)

Ans. (b)

Sol.

101. Parthyroid hormone (PTH) regulates calcium homeostasis is humans. The following statements are made regarding PRH.

A. It is a 108 amino (aa) residue long hormone whose 1-42 an exhibits full biological activity

B. It is an 84 aa hormone whose 1-34 aa exhibits full biological activity

C. An acute decreae of Ca++ results in a marked increase of PTH mRNA, followed by increased rate of PTH synthesis

D. Rate of degradation of Ca++ results in a marked when Ca++ concentrations are low

E. Cathepsin B cleaves PTH into two fragments

Which one of the following options represents the combination of all correct statements?

(a) A, C and D

(b) B, C and E

(c) A, D and E

(d) B, D and E

Ans. (b)

Sol. B. It is an 84 aa hormone whose 1-34 aa exhibits full biological activity.

C. An acute decrease of Ca2+ results in a marked increase of PTH mRNA followed by an increased rate of PTH synthesis.

E. Cathepsin B cleaves PTH into two fragments.

102. To obtain recombinant products during meiosis, a double-strand break in the DNA yields crossovers needed for chiasmata formation. The progression of the non-crossover and crossover pathways begins with the formation of D loop, however, it may not result in the production of recombinant gametes

Following statements are made regarding recombination

A. Expansion of D-loop takes place in non-crossover pathway, but not in the crossover pathway

B. Expansion of D-loop takes place in crossover pathway, but not in the non-crossover pathway

C. Ejection of elongating strand takes place in the non-crossover pathway, but not in the crossover pathway

D. Ejection of elonging strand takes place in the crossover pathway, but not in the non-crossover pathway

Which one of the following options represents the correct combination of statements that explain the formation of recombinant gametes?

(a) A and B

(b) B and C

(c) C and D

(d) D and A

Ans. (b)

Sol. Together, these statements indicate that the expansion of the D-loop in the crossover pathway and the ejection of the elongating strand in the non-crossover pathway are the keyevents involved in the formation of recombinant gametes.

103. During water stress, ABA increase dramatically in leaves causing stomatal closure. Given below are the various events involved in this process.

A. Opening of plasma membrane Ca2+ permeable ion channels and elevation of cytosolic Ca2+

B. Activation of plasma membrane anion channels, efflux of anion and potassium ions

C. Binding of ABA to cytosolic ABA receptor and inhibition of activity of Type 2C protein phosphataes (PP2Cs)

D. Phosphorylation and activation of NADPH oxidases (RBOH) and formation of apoplastic ROS.

Which one of the following options represents the correct sequence of events involved?

(a) A, B, C, D

(b) A, C, D, B

(c) C, D, B, A

(d) C, D, A, B

Ans. (d)

Sol. C. Binding of ABA to cytosolic ABA receptor and inhibition of activity of Type 2C protein phosphatases (PP2Cs). This step involves ABA binding to its receptor in the cytosol, leading to the inhibition of PP2Cs.

D. Phosphorylation and activation of NADPH oxidases (RBOH) and formation of apoplastic ROS. In this step, activated PP2Cs phosphorylate and activate NADPH oxidases, which produce reactive oxygen species (ROS) in the apoplast.

A. Opening of plasma membrane Ca2+ permeable ion channels and elevation of cytosolic Ca2+. The ROS generated in the previous step leads to the opening of plasma membrane calcium (Ca2+) permeable ion channels, resulting in an increase in cytosolic Ca2+ levels.

B. Activation of plasma membrane anion channels, efflux of anions and potassium ions. The elevated cytosolic Ca2+ triggers the activation of plasma membrane anion channels, leading to the efflux of anions and potassium ions from the guard cells, which results in stomatal closure.

104. Given below are some statements that are associated with transgenic plants. Each statement has a blank space indicated by '__________'.

A. A transgenic plant with two functional copies of a transgene can segregate in a _________ ratio for the transgenic phenotype on self-pollination if the two genes are linked.

B. The __________ system can be ued for self-pollination if the two genes are linked

C. The endogenous plant gene, _____________, can be used to engineer to resistance to imidazolinone herbicides

D. Variations in trangene expression levels between five independent trangenic lines generated using the same T-DNA contruct can be due to _____________.

Which one of the following optio has the correct sequece of terms that can be used to fill in the blanks in the above statements (from A to D) such that all statements become true?

(a) A-9:3:3:1, B-Cre/loxP, C-ALS, D-codon usage of the transgene

(b) A-3:1, B-FLP/FRT, C-bar, D-copy number of trangene

(c) A-3:1, B-Cre/loxP, C-ALS, D-position effect

(d) A-1:2:1, B-FLP/FRT, C-EPSPS, D-position effect

Ans. (c)

Sol. Cre is a 38 kDa tyrosine recombinase protein (belongs to 'integrase' family) from bacteriophage Pl which mediates site specific recombination between loxP sites. A loxP site (34 bp long) consists of two 13 bp inverted repeats separated by an 8 bp spacer or core region where recombination takes place.

The FLP-FRT system is similar to the Cre-loxP system, it involves FLP recombinase, derived from the yeast Saccharomyces cerevisiae. FLP recognizes FLP recombinase target (FRT) sequences. FRT is a 34 bp-long sequence. The site contains two inverted repeats of 13 bp each, separated by a central and asymmetric sequence of 8 bp, or spacer, which defines the orientation of the site.

105. Bacteriophage proteins, CI and Cro are crucial regulators of the lysogeny and lytic cycles of the bacteriophage. These proteins bind to the rightward operator region consisting of OR1, OR2 and OR3 (shown below).

Which one of the following statements about the regulation by CI and Cro proteins in Correct?

(a) Cro binding to OR3 activates expression of cI.

(b) CI binding to OR3 activates expression of cI ad represses the expression of cro

(c) CI bindig to OR1 and OR2 leads to repression of cI and cro

(d) CI binding to OR1 and OR2 leads to higher expression of cI and repression of cro

Ans. (d)

Sol. In lysogenic process, the C! repressor binds the operator, blocking the synthesis of Cro and also activating its own synthesis. Once C! repressor Is made from pRE, the repressor shuts off transcription from the early promoters pL and pR by binding at operators OL and O, and also controls its own synthesis from the promoter pRM. Ct repressor binds with the three operators Oal. 0,2 and 0,3. These operators have different affinities for the CI repressor, with 0,1 > 0,2 > 0,3. When there is too little Cl repressor, only the 0,1 site, which blocks the cro gene expression, is filled. The binding of the repressor to 0,1 facilitates the binding of a second molecule of repressor to 0,2. When CI repressor binds to 0,2, It serves to activate transcription of the cl gene. However, when too much Cl repressor Is present, then binding to the low-afflnlty 0,3 blocks overexpression of the cl gene

106. The following is the life table of a natural population of a small annual succulent where 'x' is its life phase, 'lx' is its surviovrship till that stage and 'dx' it its age specific mortality

(a) Seeds produced

(b) Germinated

(c) Established

(d) Rosettes

Ans. (a)

Sol. The difference between the number of individuals alive for any age (nJ and the number alive at the beginning of the next age (nlU) is the number of Individuals that have died during that time interval represent the age-specific mortality (dj. The number of individuals surviving to any given age (i.e., age specific survivorship) is calculated by taking ratio n/n0, where n0 is the number alive at the start of the study. The number of individuals that died during any given time interval (d,) divided by the number alive at the beginning of that interval (na) provides an age-specific mortality rate (qj.

107. The diagram below depicts the relationship of land plants with someof the major apomorphies indicated.

Below is a list of apomorphies that have not been labeled on the tree above

i. Intercalary growth of sporophyte

ii. Oil bodies

iii. Archegonium

iv. Leptoids

Which one of the following options correctly matches the apomorphies with their positions on the tree?

(a) A-iii, B-ii, C-iv, D-i

(b) A-i, B-iii, C-ii, D-iv

(c) A-i, B-ii, C-iii, D-iv

(d) A-ii, B-i, C-iv, D-iii

Ans. (a)

Sol. An apomorphy (or derived character) Is one that evolved in the lineage leading up to a clade and those set members of that clade apart from ancestors. A derived character shared by two or more groups is termed as synapomorphy (shared - derived - character).

108. Given below are some statements with blank spaces indicated by '__________'.

A. A plasmid cloning vector digested with an enzyme (with a single restriction site in the plasmid) that generates 3' overhangs can be made blunt-ended using ____________.

B. DNA with a nucleotide composition of 30% A, 35% G, 20% C and 15% T is most likely __________.

C. Production of only truncated molecules of transgene-derived mRNA in transgenic plants generated using a transgene from a prokaryotic source is most likely due to _________.

D. ____________ is a method for identifying the positions where individual DNA-binding proteins attach to a genome

Which one of the following options has the correct sequence of terms that can be used to complete the above statements (from A to D) such that all statements become true?

(a) A-Taq polymerase; B-single-stranded, C-presence of mRNA instability sequences, D-Fish

(b) A-Pfu plymerae; B-double stranded; C-codon usage variations; D-Chip seq

(c) A-Mung bean nuclease, B-single-stranded; C-presence of potential poly-adenylation signals in the transgene sequence; D-Chip-seq

(d) A-Reverse transcriphase, B-single-stranded, C-absence of poly A signal, D-PFGE

Ans. (c)

Sol. Statement C states that the production of only truncated molecules of transgene-derived mRNA in transgenic plants generated using a transgene from a prokaryotic source is most likely due to the presence of potential poly-adenylation signals in the transgene sequence.

109. Behavioral and cognitive responses in organisms are finely tuned to environmental cues. Given below is a list of specific hormone/chemical signals (Column X) and biological functions (Column Y).

Select the option that represents all correct matches between Column X and Column Y.

(a) A-ii, B-iv, C-i, D-iii

(b) A-iii, B-iv, C-ii, D-i

(c) A-iv, B-iii, C-i, D-ii

(d) A-iv, B-i, C-ii, D-iii

Ans. (b)

Sol. Cortisol is a hormone produced by the adrenal glands in response to stress. It plays a crucial role in the body's stress response system, helping to regulate various physiological processes.

Adrenaline, also known as epinephrine, is a hormone and neurotransmitter produced by the adrenal glands.

Melatonin is a hormone produced by the pineal gland in the brain, primarily in response to darkness.

Dopamine is a neurotransmitter that functions as a chemical messenger in the brain. It is involved in various important processes, including movement, coordination, motivation, reward, and pleasure.

110. The figure below depicts the absorption spectra of chlorophylls and carotenoid over a range of wavelengths.

Which one of the following combinations best describes A, B and C from the absorption spectra shown above?

(a) A-chlorophyll a, B-chlorophyll b, C-carotenoid

(b) A-chlorophyll a, B-carotenoid, C-chlorophyll a

(c) A-chlorophyll b, B-chlorophyll a, C-carotenoid

(d) A-carotenoid, B-chlorophyll b, C-chlorophyll a

Ans. (c)

Sol. A - chlorophyll b, B - chlorophyll a, C - carotenoid

In the absorption spectra, A corresponds to a peak that matches the absorption profile of chlorophyll b. B corresponds to a peak that matches the absorption profile of chlorophyll a. C corresponds to a peak that matches the absorption profile of carotenoid.

111. Column X lists various plant types and Column Y lists key features of these plants

Which one of the following options represents all correct matches between Column X and Column Y?

(a) A-i, B-iv, C-iii, D-ii

(b) A-iii, B-i, C-ii, D-iv

(c) A-iv, B-iii, C-i, D-ii

(d) A-ii, B-iii, C-i, D-iv

Ans. (d)

Sol. Heteroblastic: Heteroblastic refers to the phenomenon of variation in leaf shape, size, or other characteristics at different stages of a plant's life cycle. Phanerogams, also known as seed plants, are a group of plants that reproduce through the production of seeds.

Hemicryptophyte is a term used to describe a specific growth form or life strategy of plants.

Hermaphrodite refers to an organism that possesses both male and female reproductive organs or structures.

112. In yeast under anaerobic conditions, pyruvate is fermented to ethanol through two steps : decarboxylation of pyruvate to acetaldehyde and NADH-mediated reduction of acetaldehyde to ethanol. The mammalian liver also expresses alcohol dehydrogenase (Liver ADH: L-ADH). From the options given below, choose the one that best explains the physiological significance of L-ADH in the absence of fermentation in the liver

(a) The direction of L-ADH reaction varies with the relative concentrations of acetaldehyde and ethanoli. In addition, the enzyme metabolizes the alcohols produced by intestina microflora anaerobically

(b) NAD+ produced by L-ADH drives glycolysis in the liver

(c) Mammalian L-ADH converts pyruvate to lactate and the NAD+ thus generated drives glycolysis

(d) Mammalian L-ADH has non-metabolic moonlighting functions.

Ans. (a)

Sol. In the absence of fermentation, L-ADH plays a role in the metabolism of alcohols produced by intestinal microflora. It can convert acetaldehyde, which is a toxic byproduct of alcohol metabolism, into ethanol. This helps in detoxifying the body and preventing the accumulation of harmful levels of acetaldehyde. The direction of the L-ADH reaction can vary depending on the relative concentrations of acetaldehyde and ethanol. It can oxidize ethanol to acetaldehyde or reduce acetaldehyde to ethanol.

113. In wild type C. elegans hermaphrodites, two adjacent cells, Z1. ppp and Z4 aaa, have the potential to become the anchor cell. They interact in a manner that causes one of them to become the anchor cell, while the other one becomes the precursor of the uterine tissue. The following statements are given to describe the interaction of the two cells:

A. The cell secreting LAG-2 becomes the anchor cell

B. The cell secreting LIN-12 remains asthe precursor of the uterine tissue

C. The LIN-12 secreting cell takes the fate of anchor cell white the LAG-2 secreting cell takes the fate of uterine precursor cell

D. The Hippo kinase signaling pathway brings lateral inhibition so that one cell is inhibited and the other cell is promoted to become the anchor cell

Which one of the following options represents the combination of all correct statements?

(a) A and D

(b) A and B

(c) D only

(d) C only

Ans. (b)

Sol. The cell secreting LAG-2 becomes the anchor cell. C. The cell secreting LIN-12 remains as the precursor of the uterine tissue.

114. The following statements are made with regard to the optical activity of amino acids derived from natural proteins :

A. All alpha-amino acids have the D stereochemical configuration

B. All L-amino acids have the (S) absolute configration except cysteine, which has the (R) absolute configuration

C. All D-amino acids have the (S) absolute configuration except cysteine, which has the (R) stereochemical configuration.

D. In the absolute configuration system, L-threonine and L-isoleucine are (2S, 3R)-threonine and (2S, 3S)-isoleucine diastereomers, respectively

Which one of the following options represent the combinations of all correct statements?

(a) A and C

(b) B and D

(c) A and D

(d) C and D

Ans. (b)

Sol. The absolute configuration of the amino acids at the o-carbon is typically described by DL system rather than the more modern RS system. According to the RS system, all the L-amlno acids from proteins have an S absolute configuration, with the exception of L cysteine, which has R configuration. The configuration for cysteine, unlike the other chiral a-amlno adds, Is R In the RS system because of the presence of sulfur In the side chain, which raises the side-chain priority.

115. Given below are the five experiments (A-E) showing effects of duration of the light and dark periods on flowering of the short-day (SDP) and long-day plants (LDP)

Which one of the following options represents the combination of all correct flowering responses?

(a) A, B and C

(b) A, B and E

(c) B, C and D

(d) B, C and E

Ans. (b)

Sol.

116. Following statements are made about the bacterial ribosomes and their functions :

A. Association of 23S rRNA with 16 rRNA is essential to catalyze in vitro peptide bond formation using model substrates

B. The 23S rRNA is necessary and sufficient to catalyze in vitro peptide bond formation using model substratres

C. Ribosome carries a polymerization activity

D. The 16S rRNA is necessary and sufficient to catalyze in vitro peptide bond formation using model substrates.

Which one of the following options represents the combination of both correct statements?

(a) A and B

(b) B and C

(c) C and D

(d) A and D

Ans. (b)

Sol. The large 50S ribosomal subunit contains the site of catalysis—the peptidyl transferase center—which is responsible for making peptide bonds during protein elongation and for the hydrolysis of peptidyl-tRNA during the termination of protein synthesis. Peptidyl transferase activity resides in the 23S rRNA.

117. Cytoskeleton-dependent motor proteins are critical for the movement of cellular organelles in animal cells. In the fertilized egg of C. elegans, once the polarity has been established, the maternal nucleus migrates towards the paternal nucleus, which eventually leads to fusion of the two pronuclei (see below).

What molecular motor is likely to be directly involved in nuclear migration?

(a) Myosin II

(b) Kinesin

(c) Dynein

(d) Tropomyosin

Ans. (c)

Sol. Dynein is a microtubule-associated motor protein that moves towards the minus end of microtubules. It plays a crucial role in various cellular processes, including nuclear migration. Dynein interacts with microtubules and generates the force required for the movement of cellular components, such as organelles and nuclei.

118. A researcher isolated a mutant of an ER resident protein-folding enzyme (PFE) that has lost its KDEL sequence (ER retention sequence). Potential conequences of such a mutation are given below.

A. PFE is found in the extracellular space

B. PFE is degraded in the ER

C. Unfolded proteins increase in the ER

D. DFE is transported to the cytosol

Which one of the following options represents the combination of all correct statements?

(a) B and C

(b) A and D

(c) A and C

(d) C and D

Ans. (c)

Sol. The KDEL sequence, which acts as sorting signal is both necessary and sufficient for retention In the ER. If this ER retention signal is removed from BiP, for example, the protein is secreted from the cell; and if the signal is transferred to a protein that Is normally secreted, the protein Is now retained in the ER.

119. The figure below represents the data from immuoblots (IB) of co-immunoprecipitation (IP) experiments with antibodies as indicated, after HeLa cells were treated witharsenic (-cat refers to -catenin; and E-cad refers to E-cadherin).

The following assumptions are made :

A. Arsenic activates the non-canonical -catenin pathway

B. Arsenic leads to cellular proliferation

C. Arsenic leads to proteasomal degradation of -catenin

D. Arsenic induces apoptosis in the HeLa cells

Which one of the following options represents the combination of all correct assumptions?

(a) A and B

(b) B and C

(c) C and D

(d) A and D

Ans. (a)

Sol. Assumption A states that arsenic activates the non-canonical -catenin pathway. Assumption B states that arsenic leads to cellular proliferation.

120. In an experiment, immunophenotyping of human blood cells was done to analyse the relative number of CD8+ T cells and CD4+ Tcells. The following antibody-fluorchrome conjugates for staining different blood cells were available

i. CD19-FITC for B lymphocytes

ii. CD8-CY7 PE for T lymhocytes

iii. CD4-Cy7 PE for T lymphocytes

iv. CD3-PE for T lymphocytes

v. DAPI for nucleus

Which one of the following options gives the correct sequence of antibody-fluorochrome conjugates, to be used to sort the relative abundance of CD8+ ad CD4+ cells in the given blood sample by FACS?

(a) DAPI and CD3-PE

(b) DAPI : CD3-PD; CD8-CY7 PE or CD-Cy7 PE

(c) CD19-FITC and CD3-PE; CD8-CY7 PE or CD4-Cy7 PE

(d) DAPI CD19-FITC and CD3-PE; CD8-CyY7 PE and CD4-Cy7 PE

Ans. (d)

Sol. DAPI; CD19-FITC and CD3-PE; CD8-CY7 PE and CD4-Cy7 PE

This means that we should use DAPI for nucleus staining, CD19-FITC and CD3-PE for B and T lymphocytes, and both CD8-CY7 PE and CD4-Cy7 PE for T lymphocytes. By using this combination, we can differentiate and sort the relative abundance of CD8+ and CD4+ cells in the blood sample using FACS.

121. The following statements are made regarding the characteristic features of body temperature in humans:

A. The core body temperature varies least with the changes of environmental temperature

B. During severe mucular exercise the rectal temperature may rise up to 40°C

C. The oral temperature is relatively higher than the rectal temperature

D. The core body temperature is highest at 6:00 AM and lowest in the evening in humans who sleep at night and remain awake during day time

E. The temperature of scrotum is regulated at 37°C

F. In women, a rise of basal body temperature occurs immediately after ovulation.

Which one of the following options represents the Incorrect combination of the statements?

(a) A, B, C

(b) B, C, D

(c) C, D, E

(d) D, E, F

Ans. (c)

Sol. Statement C says that the oral temperature is relatively higher than the rectal temperature. This is incorrect because the rectal temperature is generally higher than the oral temperature.

Statement D says that the core body temperature is highest at 6:00 AM and lowest in the evening in humans who sleep at night and remain awake during the daytime. This isincorrect because the core body temperature follows a circadian rhythm, where it is typically lowest in the early morning (around 4:00 AM) and highest in the late afternoon or early evening.

Statement E says that the temperature of the scrotum is regulated at 37°C. This is incorrect because the scrotum is responsible for regulating the temperature of the testes, not maintaining a constant temperature of 37°C.

122. The following statements are made regarding materials transported through the phloem of a plant.

A. Only reducing sugars are translocated

B. Non-reducing sugars are generally translocated

C. Sucrose and raffinose are generally translocated

D. Only D-glucose and D-fructose are translocated

Whichn one of the following options represents the combination of all correct statements?

(a) A, C and D

(b) B and C only

(c) B and D only

(d) A and C only

Ans. (b)

Sol. The translocated carbohydrates are mostly nonreducing sugars and sugar alcohols (sorbitol and mannitol). Sucrose Is the most common translocated sugar. Sucrose Is a dlsaccharide made up of one glucose and one fructose molecule. Some plant families. In addition to sucrose, translocate oligosaccharides of raffinose family. Members of this family Includes raffinose (trisaccharide), stachyose (tetrasaccharide), and verbascose (pentasaccharide).

123. Column X lists evoutionary ideas and scientists who proposed them, and Column Y lists the description the these ideas

Which one of the following options represents all correct matches between Column X and Column Y?

(a) A-iii, B-iv, C-ii, D-i

(b) A-i, B-ii, C-iv, D-iii

(c) A-ii, B-iv, C-iii, D-i

(d) A-iv, B-i, C-ii, D-iii

Ans. (a)

Sol. The Modern Synthesis combined Darwin's theory of evolution through natural selection with the understanding of genetic inheritance, providing a comprehensive framework for understanding the mechanisms of evolutionary change.

According to this idea, species evolve through slow and incremental modifications in their characteristics, gradually transitioning from one form to another.

It suggests that evolutionary change often occurs in relatively brief periods of rapid speciation (punctuation) followed by long periods of stability (equilibrium) in the fossil record.

The coalescent model allows researchers to understand the patterns of genetic variation within populations and estimate the time to the most recent common ancestor.

124. The following statements are made regarding male reproductive system, particularly with reference to spermatogenesis and sperm production

A. The membrane of spermatozoa contain germinal angiotensin-coverting enzyme (gACE).

B. Mature spermatozoa are released from Leydig cells

C. Sertoli cells secrete Mullerian inhibiting substance (MIS)

D. Sertoli cells synthesize androgens

E. Rate testis has high content of estrogen and alpha estrogen receptors

Which one of the following options represents the combination of all correct statements?

(a) A, B and C

(b) B, C and D

(c) C, D and E

(d) A, C and E

Ans. (d)

Sol. Sertoli cells, which are found In the seminiferous tubules of the testes, secrete Mullerian inhibiting substance (MIS). MIS is responsible for the regression of the Mullerian ducts during embryonic development, leading to the formation of male reproductive structures. The rete testis is a network of tubules located in the testes. It has a high content of estrogen and alpha estrogen receptors (ERa). Estrogen receptors are involved in the response to estrogen hormones.

125. Columns X and Y of the following table list some treatement methods, reagents, and events that are related to human lymphocyte culture, and banding/karyotyping of human chromosomes.

Which one of the following options represents all correct matches between Column X and Column Y?

(a) A-ii, B-v, C-iii, D-i, E-iv

(b) A-v, B-iii, C-ii, D-iv, E-i

(c) A-iv, B-v, C-i, D-iii, E-ii

(d) A-ii, B-v, C-iv, D-i, E-iii

Ans. (a)

Sol.

126. Column X lists proteins that play a role in mediating DNA recombination processes and Column Y lists the possible functions of these proteins.

Which one of the following options represents all correct matches between Column X and Column Y?

(a) A-i, B-ii, C-iv, D-iii

(b) A-iv, B-i, C-ii, D-iii

(c) A-iv, B-iii, C-i, D-ii

(d) A-iii, B-iv, C-ii, D-i

Ans. (c)

Sol. RAD51 is a protein that plays a crucial role in DNA repair by homologous recombination (HR).

SPO11 is an enzyme involved in the formation of double-strand breaks during meiosis, a specialized cell division process that produces haploid gametes (e.g., eggs and sperm) in sexually reproducing organisms.

RAD52 interacts with other proteins involved in the HR pathway and facilitates the exchange of genetic material between DNA molecules.

RAD59 is a protein involved in the repair of DNA double-strand breaks through a pathway called single-strand annealing (SSA).

MRX, on the other hand, refers to a complex of three proteins: Mre11, Rad50, and Xrs2 (Nbs1 in humans).

127. Given below are the approximate lengths of DNA fragments obtained on agaroe gel electroporesis following restriction digestion of a 3kb circular plasmid with different restriction enzymes:

BamHI : 0.5kb, 2.5kb

HincII : 3kb

EcoRI : 3kb

EcoRI + BamHI : 0.5kb, 1kb, 1.5kb

EcoRI + BamHI : 0.5kb, 1kb, 1.5kb

EcoRI + HincII : 1.3kb, 1.7kb

BamHI + HincII : 0.2kb, 0.3kb, 2.5kb

Based on the above information, which one of the following statements is Incorrect?

(a) HincII and EcoRI have a single recognition site each in the plasmid

(b) HincII site is located between two BamHI sites

(c) The distance between EcoRI and BamHI sites is less than that between the HincII site and BamHI sites

(d) HincII is located closer to one BamHI site than the other

Ans. (c)

Sol. The distance between EcoRI and BamHI sites is less than that between the Hindi site and BamHI sites.

The information provided indicates that the lengths of the DNA fragments obtained after restriction digestion with EcoRI and BamHI are the same (3 kb), suggesting that the EcoRI and BamHI sites are adjacent or overlapping. Therefore, the distance between EcoRI and BamHI sites would be less than the distance between the Hindi site and BamHI sites..

128. Many species of birds cell at drawn in temperature regions. The phenomenon is referred to as "Dawn Chorus". Several explanations have been proposed for this. Which one of the options is NOT a correct explanation for the occurrence of "Dawn Chorus"?

(a) Transmission of sound is better at drawn due to colder temperature at that time

(b) Singing at dawn is costly as the birds are low on energy. This makes singing at dawn a handicap and thereby indicates honet signalling

(c) Dawn chorus allows birds to utilie a time window for singing which does not interfere with their feeding time

(d) The syrinx muscles are unable to move freely after early morning, resulting in poorer control over song production at later times of the day

Ans. (d)

Sol. The syrinx is the vocal organ in birds responsible for producing sounds, including bird calls. The activity of the syrinx muscles is indeed involved in bird vocalizations. However, it is not a direct explanation for the occurrence of "Dawn Chorus." The "Dawn Chorus" phenomenon is primarily attributed to other factors such as increased territorial behavior, mating displays, and communication among birds during the early morning hours.

129. Shown below are the CD spectra of a protein recorded under two different conditions

From the options given below, select thne one that is the best interpretation of the spectra.

(a) The protein has helical secondary structure under condition A that is denatured under condition B.

(b) The protein has heical secondary structure under condition A that is converted to sheets under condition B.

(c) The spectra represent the tertiary fold of the protein with condition A corresponding to mixed helical + sheet fold and condition B corresponding to largely β sheet fold

(d) The difference between the spectra under conditions A and B is due to lower protein concentration under condition B

Ans. (a)

Sol. Based on the given information, the CD spectra of the protein under two different conditions are compared. The spectra show a difference between condition A and condition B.

In option a, it is stated that the protein has a helical secondary structure under condition A, which means that the CD spectrum of condition A indicates the presence of helical structure in the protein. However, under condition B, the spectrum suggests denaturation of the protein, indicating the loss or disruption of the helical structure.

130. The two phylogentic trees given below represent evolutionary patterns in species or population. The differently colored or dashed lines represent a single species or gene genealogy

Select the option that correctly identifies the type of evolutionary process that these two figures represent

(a) A-hybridization, B-incomplete lineage sorting

(b) A-convergence, B-incomplete lineage sorting

(c) A-adaptive introgression, B-hybridization

(d) A-hybridization, B-adaptive introgression

Ans. (a)

Sol. Hybridization is a process where two different species interbreed and produce offspring. It involves the exchange of genetic material between two distinct species, leading to the formation of hybrid individuals. This process can occur when two species come into contact and successfully reproduce.

131. Two batches of antibodies (Q and R) were generated for an antigen and affinities of both the antibodies were assayed using pure antigen. Given below are Scatchard plots obtained for the antibody-antigen binding assays and the inferences drawn upon Scatchard analysis.

A. Antibody Q is possibly a monoclonal white R is polyclonal

B. The curved nature of Scatchard plot for R indicates that it cross-reacts with the blocking reagent

C. The average affinity of R is more than affinity of Q to the antigen

D. Antibody Q is possibly IgA and R is IgF

E. The valency of the antibodies cannot be inferred from thne Scatchard plots.

Select the option that groups all the correct inferences.

(a) A, B, C

(b) B, D, E

(c) A, C, D

(d) B, C, E

Ans. (c)

Sol. A. Antibody Q is possibly a monoclonal while R is polyclonal. This inference suggests that antibody Q may be derived from a single clone, while antibody R may be derived from multiple clones.

C. The average affinity of R is more than the affinity of Q to the antigen. This inference indicates that, on average, antibody R has a higher affinity for the antigen compared to antibody Q.

D. Antibody Q is possibly IgA and R is IgG. This inference suggests that antibody Q may belong to the IgA class, while antibody R may belong to the IgG class.

132. A mutant DNA polymerase was found to have higher error rate and synthesized only short DNA fragments. In the statements below, potential explanations are given.

A. The 5' to 3' exonuclease activity is compromised

B. The 3' to 5' exonuclease activity is compromised

C. The polymerase tends to frequently dissociate from the template

D. The polymerase is unable to unwind the DNA template during replication

Which one of the following options represents the combination of all correct statements?

(a) A and B

(b) C and D

(c) A and D

(d) B and C

Ans. (d)

Sol. Statement B suggests that the 3' to 5' exonuclease activity is compromised in the mutant DNA polymerase. Exonuclease activity allows for proofreading and removal of incorrectly incorporated nucleotides during DNA synthesis, so a compromised 3' to 5' exonuclease activity would result in a higher error rate.

Statement C suggests that the polymerase tends to frequently dissociate from the template. This would explain why the mutant DNA polymerase synthesizes only short DNA fragments since frequent dissociation would prevent the synthesis of longer DNA strands.

133. The theory of island biogeography has synthesized into theory the following concepts, except:

(a) Competition

(b) Immigration

(c) Equilibrium

(d) Speciation

Ans. (d)

Sol. One concept that is not directly synthesized into this theory is speciation. The theory primarily focuses on immigration and extinction rates as the primary factors influencing species richness on islands. Speciation, which refers to the formation of new species, is not a central component of the theory of Island biogeography.

134. The following represents selected AFLP bands (I to V) observed in parents (P1 and P2), F1 progeny and 20 doubled haploid (DH) progeny developed from the F1. DH are created through chromosome doubling of pollen grains in anther culture.

The following statements were made about the above AFLP bands :

A. Bands I and IV are allelic

B. Bands II and V assort independently

C. Band III is uninformative

Which one of the following options represents a combination of all correct statements?

(a) A only

(b) C only

(c) A and B only

(d) A, B and C

Ans. (d)

Sol. A. Bands I and IV are allelic. This statement suggests that bands I and IV are derived from the same genomic locus but represent different alleles. In other words, they are alternative forms of the same gene or DNA sequence.

B. Bands II and V assort independently. This statement implies that the presence orabsence of band II does not affect the presence or absence of band V, and vice versa. They segregate independently during inheritance and are not linked.

C. Band III is uninformative. This statement suggests that band III does not provide useful orin formative data for genetic analysis. It could be a result of technical artifacts, non-polymorphic regions, or other factors that make it less relevant for studying genetic variation.

135. Two protein kinase, K1 and K2 regulate an intracellular pathway in response to the extracellular signal. The following observations were made regarding the pathway.

i. Response is observed even in the absence of an extracellular signa when a mutation permanently activates K1

ii. Response is observed even in the absence of an extracellular signal when K1 contains an activating mutation and K2 has an inactivating mutation

iii. No response is observed even in the presence of an extracellular signal when K2 is inactivated by mutations

iv. Response is observed even in the absence of an extracellular signal when both kinases are activated by mutations

Which one of the following statements is correct?

(a) K1 inhibits K2

(b) K2 inhibits K1

(c) K1 activates K2

(d) K2 activates K1

Ans. (d)

Sol. Observation I suggests that the activation of K1 alone is sufficient to induce a response in the absence of an extracellular signal. This indicates that K1 plays a crucial role in the pathway.

Observation II suggests that even when K1 contains an activating mutation, a response is still observed in the absence of an extracellular signal if K2 is inactivated. This indicates that K2 acts as a negative regulator and its inactivation allows the pathway to proceed.

Observation III suggests that when K2 is inactivated by mutations, no response is observed even in the presence of an extracellular signal. This further supports the role of K2 in the pathway.

136. Following statements with respect to development in sea urchin were put forth :

A. The cell fates are determined both by autonomous and conditional modes of specifaction.

B. Large micromeres are conditionally specified

C. Large micromeres produce paracrine and juxtacrine factors that specify the fates of their neighbours

D. -catenin is not required for the specification of the micromeres

Which one of the following options represents the combination of all correct statements?

(a) A and B

(b) A and C

(c) C and D

(d) B and D

Ans. (b)

Sol. The cell fates are determined by both autonomous and conditional modes of specification. The large mlcromeres are autonomously specified. The autonomously specified large mlcromeres are now able to produce paracrine and juxtacrlne factors that conditionally specify the fates of their neighbors.

A key signal transduction pathway influencing fate of blastomeres is the canonical Wnt signaling pathway Signaling through the Wnt pathway is necessary for polarity establishment, patterning, and germ layer specification in the sea urchin embryo. Maternal p-catenin is important for the autonomous specification of the micromeres.

137. Two mutations were isolated in bacteriophage, one causing clear plaque (c) and the other causing minute plaque (m). The genes responsible for these two mutations are 9 cM apart. The plaques with genotype c+ m and c m+ were mixed to infect bacterial cells. The progeny plaques were collected, cultured and plated on bacteria.

The expected number of the different types of plaques are shown below

A. c+ m+ 455, c+ m 45, c m+ 45, c m 455

B. c+ m+ 455, c+ m 455, c m+ 45, c m 45

C. c+ m+ 45, c+ m 455, c m+ 455, c m 45

D. c+ m+ 65, c+ m 680, c m+ 685, c m 70

Which one of the following options represents the combination of all correct statements?

(a) A only

(b) B only

(c) C only

(d) C and D

Ans. (d)

Sol. Based on the information provided, option C and D are the only options that match the expected number of different types of plaques observed after mixing and infecting bacterial cells.

Option C (c* m* 45, c* uf 455, c* m* 455, c* tu* 45) represents the expected number of plaques with specific genotypes, including clear plaque (c*) and minute plaque (m*).

Option D (c* m* 65, c* m' 680, c' m* 685, c' m' 70) also represents the expected number of plaques with specific genotypes, including clear plaque (c*) and minute plaque (m*).

138. Select the rare or endangered species which also have exceptionally low genetic variability, as documented by multi-locus molecular methods.

(a) Eucalyptus phylacis (Australian Meelup Mallee)

(b) Impatiens parviflora (Small balsam)

(c) Pavo cristatus (Indian peacock)

(d) Hydrobates castro (Hawaiian Band-rumped Stom Petrel)

Ans. (a)

Sol. Multi-locus molecular methods refer to techniques that analyze multiple genetic loci or markers simultaneously to assess genetic variability within a population.

139. In Droophila, a cross was set between a male homozygous for alleles s+/s+ (pheotype A) and a female homozygous of s/s (phenotype B) ('s+' being a dominant allele and 's' a recessive allele). All of the F1 individuals thus obtained had the phenotype B. When F1 individuals were crossed among themselves, all progeny obtained were of phenotype A in F2.

The following explanations were proposed for the results obtained

A. This is an example of cytoplamic inheritance

B. This is exhibiting genetic maternal effect

C. This is a quantitative trait influenced by the environment

D. This is exhibiting gene interaction with epistasis

E. The trait is showing position effect variegation

Which one of the following option is correct?

(a) A only

(b) B only

(c) C only

(d) D and E

Ans. (b)

Sol. There is a difference between maternal inheritance and maternal effect. Maternal inheritance occurs when the hereditary determinants of a trait are extranudear and genetic transmission Is only through the maternal cytoplasm, whereas the moternol effect occurs when the nuclear genotype of the mother determines the phenotype of progeny. In maternal effect, the hereditary determinants are nuclear genes transmitted by both sexes.

140. Given below are a few statements on mapping populations and marker-assisted selection (MAS):

A. MAS can be used to eliminate undersirable genotypes early in the breeding progem by screening plants at the seeding stage

B. In backcross breeding programs, breeders use molecular markers to select against the donor genome to accelerate recovery of the recurrent parent genome

C. Among different types of mapping populations, F2 and F2:3 populations are immortal populations

D. Near Isogenic Lines (NILs) can be produced by repeated self-pollination of F1.

Which one of the following options represents the combination of all correct statements?

(a) A and D

(b) B and D

(c) C and A

(d) A and B

Ans. (d)

Sol. Molecular plant breeding Is the applications of biotechnology to improve or develop new cultivars, which includes two major approaches, marker-assisted selection (MAS) and genetic transformation. Marker-assisted selection refers to the use of molecular markers to assist phenotypic selections in crop improvement. In this process, molecular markers areused for the indirect selection on traits of interest in crops. As a critical and effective method, MAS has been widely applied in plant breeding to enhance crop yield, quality, and tolerance to biotic or abiotic stresses.

141. Which of the following plots best depicts growth as per the logistic equation?

(a)

(b)

(c)

(d)

Ans. (c)

Sol.

142. Following statements are made about fertilization occurring in sea urchins:

A. Chemottraction of the sperm to the egg is mediated by sperm activating peptides like bindin

B. Exocytosis of the sperm acrosomal vesicles and release of enzymes occur

C. The capacitated sperm undergoes acrosome reaction

D. The acrosome protein mediating the critical species-specific binding event in resact

E. The slow block to polyspermy is accomplished by the cortical gramule reaction

Which one of the following options represents the combination of all correct statements?

(a) A and B only

(b) A, B and D

(c) B and E only

(d) A, B and E

Ans. (c)

Sol.

143. The following statements are potential explanations for the continued existence of genes that control eye development in eyeless cavefish

A. They have inherited these genes from their ancestors and his remains even though they no longer have eyes

B. In case of a possibility that they return to the surface environment retention of vision would be advantageous, so evolution retains this trait.

C. Evolution can only lead to gain of a trait, no loss of a trait

D. These genes are retained because of combined role of these genes with other sensory mechanisms

Which one of the following options represents the combination of correct statements?

(a) A and B

(b) B and C

(c) A and D

(d) C and D

Ans. (c)

Sol. Statement A suggests that eyeless cavefish have inherited the genes controlling eye development from their ancestors, even though they no longer have functional eyes. This is a possible explanation for the continued existence of these genes in cavefish populations. Statement D suggests that these genes may have a combined role with other sensory mechanisms. While the cavefish may have lost functional eyes, these genes may still play a role in other sensory processes or developmental pathways.

144. The poly below has two curves (A, B) that show the fractional occupancy of hemoglobin and myoglobin by oxygen as a function of the amount of oxygen.

The two reactions are

i. E + S ES and

ii. E = nS ESn.

where S is O2 and E is myoglobin or hemoglobin

The equations that could be used to fit the two curves are :

II. where K = ([E][S]n)/[ESn]

III.

IV. where K = ([E][S])/[ES]

YO2 is the fraction of oxygen-binding sites occupied by oxygen pO2 is partial pressure of oxygen.

From the options given below, select the option with the right curve (A, B), reaction (i, ii) and equations (I, II, III, IV) that describe oxygen binding to hemoglobin and myolglobin.

(a) Myoglobin : curve A, reaction i, equations III and IV. Hemoglobin : curve B, reaction ii, equations I and II.

(b) Myoglobin : curve B, reaction i, equations II and IV. Hemogobin curve : A, reaction ii, equation I and III

(c) Myoglobin curve A, reaction ii, equations III and IV. Hemoglobin: curve B, reaction i, equations I and II.

(d) Myoglobin : curve A, reaction ii, equations I and II. Hemoglobin curve B, reaction i, equations III and IV.

Ans. (a)

Sol. Myoglobin: curve A (describing oxygen binding), reaction I, equations III and IV. Hemoglobin: curve B (describing oxygen binding), reaction II, equations I and II.

This indicates that for myoglobin, the oxygen binding curve is represented by curve A. The reaction involved is reaction I, and the equations describing the binding process are equations III and IV. For hemoglobin, the oxygen binding curve is represented by curve B.

The reaction involved is reaction II, and the equations describing the binding process are equations I and II.

145. The following statements were made about Laser Scanning Confocal Microscopy (LSCM).

A. LSCM is a wide field, technique with Kohler illumination system

B. Spatial resolution higher than that achieved in wide field imaging could be obtained if only the central portion of an Airy Disk is used to form an image

C. Scanning mirrors sweep the excitation beam over the sample point-by-point to build the image

D. An altered pinhole size does not make any impact on the resolution of the image

E. A photomultiplier tube (PMT) in LSCM helps in generating real colour of fluorophores

Which one of the following options represents the combination of all correct statements?

(a) A, B and D

(b) C, D and E

(c) B and C only

(d) B and E only

Ans. (c)

Sol. Statement B is correct. In Laser Scanning Confocal Microscopy (LSCM), spatial resolution higher than that achieved in wide field imaging can be obtained by using only the central portion of an Airy Disk to form an image. This allows for improved resolution and reduction of out-of-focus light.

Statement C is also correct. LSCM uses scanning mirrors to sweep the excitation beam over the sample point-by-point, building the image as it scans. This scanning process allows for the collection of optical sections at different depths within the sample.